Mathcenter Forum

Mathcenter Forum (https://www.mathcenter.net/forum/index.php)
-   อสมการ (https://www.mathcenter.net/forum/forumdisplay.php?f=18)
-   -   Inequality Marathon (https://www.mathcenter.net/forum/showthread.php?t=1186)

devilzoa 04 มีนาคม 2007 21:33

28.สมมติให้ $a=\frac{x}{y},b=\frac{y}{z},c=\frac{z}{x} , a,b,c>0$
ใช้ Am-Gm จะได้ $a+b+c\ge3$ จะได้ว่า $\frac{(a+b+c)^{2}}{3}\ge a+b+c$
ใช้ power mean ${a^{2}+b^{2}+c^{2}}\ge\frac{(a+b+c)^{2}}{3}\ge a+b+c$

**มีสั้นกว่านี้ไหมครับ ชี้แนะด้วยครับ ผมยังมือใหม่อยู่ \_/?

nooonuii 05 มีนาคม 2007 00:54

อ้างอิง:

ข้อความเดิมของคุณ devilzoa:
28.สมมติให้ $a=\frac{x}{y},b=\frac{y}{z},c=\frac{z}{x} , a,b,c>0$
ใช้ Am-Gm จะได้ $a+b+c\ge3$ จะได้ว่า $\frac{(a+b+c)^{2}}{3}\ge a+b+c$
ใช้ power mean ${a^{2}+b^{2}+c^{2}}\ge\frac{(a+b+c)^{2}}{3}\ge a+b+c$

**มีสั้นกว่านี้ไหมครับ ชี้แนะด้วยครับ ผมยังมือใหม่อยู่ \_/?


ไม่รู้ว่าสั้นรึเปล่านะครับ แต่ผมมีอีกสองวิธี

First Solution :
โดย AM-GM จะได้ $a^2+b^2+c^2\geq 3$
โดยอสมการโคชีจะได้ $a+b+c\leq\sqrt{3(a^2+b^2+c^2)}\leq a^2+b^2+c^2$

Second Solution :
$$(a^2-a)+(b^2-b)+(c^2-c)=(a-1)^2+(b-1)^2+(c-1)^2+(a+b+c-3)\geq 0$$

nooonuii 05 มีนาคม 2007 08:52

มาเติมโจทย์ครับ

31. $a,b,c>0$
$$\frac{a}{b+c}+\frac{b}{c+a}+\frac{c}{a+b}\geq\frac{3}{2}$$

32. $a,b,c,d>0$

$$\frac{a}{b+c}+\frac{b}{c+d}+\frac{c}{d+a}+\frac{d}{a+b}\geq 2$$

33. $a,b,c,d\geq 0$
$$\sqrt{(ad+bc)(ab+cd)}\leq \frac{(a+b+c+d)^2}{8}$$

34. $a,b,c\geq 0$
$$a^4+b^4+c^4\geq abc(a+b+c)$$

35. $a,b,c>0$
$$a^bb^cc^a\leq \Big(\frac{a+b+c}{3}\Big)^{a+b+c}\leq a^ab^bc^c$$

devilzoa 05 มีนาคม 2007 15:50

ข้อ 31 Chebyshev's inequality และ Am-Hm
$3(\frac{a}{b+c}+\frac{b}{a+c}+\frac{c}{a+b})\ge (a+b+c)(\frac{1}{b+c}+\frac{1}{a+c}+\frac{1}{a+b})\ge\frac{9}{2}$
จะได้ $\frac{a}{b+c}+\frac{b}{a+c}+\frac{c}{a+b}\ge\frac{3}{2}$

ข้อ 32 น่าจะทำแบบเดียวกันกับข้อ 31
ข้อ 33 ใช้ Am-Gm $\frac{(ad+bc)+(ab+cd)}{2}=\frac{(a+c)(b+d)}{2}\ge\sqrt{(ad+bc)(ab+cd)}$
และ $\frac{(a+b+c+d)^{2}}{4}\ge(a+c)(b+d)$
ดังนั้น $\frac{(a+b+c+d)^{2}}{8}\ge\frac{(a+c)(b+d)}{2}\ge\sqrt{(ad+bc)(ab+cd)}$

ข้อ 34 Am-Gm $(\frac{a+b+c}{3})^{3}\ge abc$ ดังนั้น $\frac{(a+b+c)^4}{(3)^{3}}\ge abc(a+b+c)$
Power mean $a^{4}+b^{4}+c^{4}\ge\frac{(a+b+c)^4}{(3)^{3}}\ge abc(a+b+c)$

nooonuii 06 มีนาคม 2007 09:34

31. Alternative Solution:
ใช้อสมการโคชีจะได้
$$ab+bc+ca\leq a^2+b^2+c^2 \Rightarrow 3(ab+bc+ca)\leq (a+b+c)^2$$

ใช้อสมการโคชีอีกครั้งจะได้ว่า

$\begin{array}{rcl} a+b+c & = & \displaystyle{ \sqrt{\frac{a}{b+c}}\cdot \sqrt{a(b+c)}+\sqrt{\frac{b}{c+a}}\cdot \sqrt{b(c+a)}+\sqrt{\frac{c}{a+b}}\cdot \sqrt{c(a+b)} } \\ & \leq & \displaystyle{ \sqrt{\frac{a}{b+c} + \frac{b}{c+a}+\frac{c}{a+b}} \sqrt{2(ab+bc+ca)}} \\
& \leq & \displaystyle{ \sqrt{\frac{a}{b+c} + \frac{b}{c+a}+\frac{c}{a+b}} \sqrt{\frac{2}{3}(a+b+c)^2} } \\
& = & \displaystyle{ \sqrt{\frac{2}{3}\Big(\frac{a}{b+c} + \frac{b}{c+a}+\frac{c}{a+b}\Big)} \Big(a+b+c \Big) } \end{array}$

จัดรูปจะได้อสมการตามต้องการ :eek:

32. ไม่น่าจะใช้ Chebychev's inequality ได้ครับ เพราะจะมีปัญหาตอนเรียงค่า

34. Alternative Solution:
ใช้อสมการโคชีสองครั้ง :D
$\begin{array}{rcl} abc(a+b+c) & = & ab\cdot ca + bc\cdot ab + ca\cdot bc \\ & \leq & a^2b^2+b^2c^2+c^2a^2 \\
& \leq & a^4+b^4+c^4 \end{array}$

Punk 05 พฤษภาคม 2007 06:03

ไม่ได้จะมาทำโจทย์แต่มาเพิ่มโจทย์ครับ ข้อนี้ไม่ยากเผอิญไปได้แนวคิดมาจากการพิสูจน์ isodiametric inequality ครับ

36. ให้ $E$ เป็นเซตของจุดบนระนาบ ซึ่งสมมาตรกับจุดกำเนิด และระยะระหว่างสองจุดใดๆไม่เกิน $d$ จงพิสูจน์ว่าพื้นที่ของเซต $E$ สอดคล้องอสมการ
\[
Area(E)\leq\pi\frac{d^2}{4}
\]

warut 05 พฤษภาคม 2007 07:12

ผมว่า ข้อ 36. เป็นโจทย์ที่ง่ายที่สุดที่คุณ Punk เคยเอามาโพสต์เลยนะครับ (ทุกทีแค่อ่านโจทย์อย่างเดียว ก็ยังยากที่จะเข้าใจเลย)

ให้ $(x,y)\in E$ ดังนั้น $(-x,-y)\in E$ ด้วย เนื่องจาก $E$ มีสมมาตรกับจุดกำเนิด

ให้ $x^2+y^2=r^2$ โดยที่ $r\ge0$ เราจะได้ระยะทางระหว่าง $(x,y)$ กับ $(-x,-y)$ คือ $2r$ ดังนั้น $2r\le d$

แสดงว่า $x^2+y^2\le \left( \frac{d}{2} \right)^2$ เราจึงได้ว่า $E\subseteq \{ (x,y)\in \mathbb R^2 \mid x^2+y^2\le \left( \frac{d}{2} \right)^2 \}$ และ $$Area(E)\le\pi\frac{d^2}{4}$$

Punk 05 พฤษภาคม 2007 08:36

แฮะๆๆๆคิดอยู่แล้วเชียวว่าต้องเรียบร้อย(โรงเรียน)คุณ warut :p ข้อนี้ผมว่ามัน tricky นิดหน่อยนะครับ แบบที่เค้าว่ากันว่าคณิตศาสตร์บางครั้งต้องตีลังกาคิด :)

nooonuii 05 พฤษภาคม 2007 09:01

ในที่สุดความลับสวรรค์ก็เปิดเผย มิน่าผมคิดเท่าไรก็คิดไม่ออก เราต้องตีลังกาคิดนี่เองครับ :laugh: :laugh:

ป.ล. พี่ Punk ไม่ส่งรูปมาโชว์บ้างเลยครับ จะได้เก็บไว้เป็นที่ระลึก :)

kanakon 19 พฤษภาคม 2007 09:07

เห็นกระทู้เงียบๆ เลยเอามาฝากครับ

37. If a,b,c,...,k are +ve quantities. Prove that

$$\left(\frac{a+b+c+...+k}{n} \,\right)^{a+b+c+...+k}<{a^a}{b^b}{c^c}...{k^k}$$

nooonuii 19 พฤษภาคม 2007 09:49

อ้างอิง:

ข้อความเดิมเขียนโดยคุณ kanakon (ข้อความที่ 19028)
เห็นกระทู้เงียบๆ เลยเอามาฝากครับ

37. If $a_1,a_2,...,a_n$ are positive quantities. Prove that

$$\left(\frac{a_1+a_2+\cdots+a_n}{n} \,\right)^{a_1+a_2+\cdots+a_n} \leq {a_1^{a_1}}{a_2^{a_2}}\cdots {a_n^{a_n}}$$


kanakon 01 สิงหาคม 2007 22:37

ก่อนสอบรอบสองผมขอทิ้งโจทย์ประดับบอร์ดอีกข้อนะครับ
เพราะบอร์ด อสมการ เงียบเชียบเหลือเกินไม่ค่อยมีคนสนใจเลย

Let $n\geq 3$ be an integer. Let $t_1,t_2,t_3,...t_n$ be posiive real integer such that
$$(n^2+1)>(t_1+t_2+...+t_n)(\frac{1}{t_1}+\frac{1}{t_2}+...+\frac{1}{t_n} )$$
Show that $t_i,t_j,t_k$ are side lenghts of a triangle for all $i,j,k$ with $1\leq i<j<k\leq n$ [IMO 2004]

Spotanus 27 สิงหาคม 2007 00:32

เพิ่งคิดโจทย์ตะกี้เอง พอดีเห็นคุณ nooonuii บอกให้ลองมาเล่นดูครับ
37.อสมการ
:)

tatari/nightmare 29 สิงหาคม 2007 20:50

อ้างอิง:

ข้อความเดิมเขียนโดยคุณ kanakon (ข้อความที่ 21315)
Let $n\geq 3$ be an integer. Let $t_1,t_2,t_3,...t_n$ be posiive real integer such that
$$(n^2+1)>(t_1+t_2+...+t_n)(\frac{1}{t_1}+\frac{1}{t_2}+...+\frac{1}{t_n} )$$
Show that $t_i,t_j,t_k$ are side lenghts of a triangle for all $i,j,k$ with $1\leq i<j<k\leq n$ [IMO 2004]

$(t_1+t_2+...+t_n)(\frac{1}{t_1}+\frac{1}{t_2}+...+\frac{1}{t_n})=n+\sum_{i<j}^{}(\frac{t_i}{t_j}+\frac{t_j}{t_i})
=n+t_1(\frac{1}{t_2}+\frac{1}{t_3})+\frac{1}{t_1}(t_2+t_3)+\sum_{i<j,(i,j)\not\in{(1,2),(1,3)}}^{}(\frac{t_i}{t_j}+\frac{t_j}{t_ i})$
จาก $$\frac{1}{t_2}+\frac{1}{t_3}\geq\frac{2}{\sqrt{t_{2}t_{3}}}$$
$$t_2+t_3\geq 2\sqrt{t_{2}t_{3}}$$ ( AM-GM)
และ $\frac{t_i}{t_j}+\frac{t_j}{t_i}\geq2$ ให้ $a=\frac{t_1}{\sqrt{t_{2}t_{3}}}$ ได้ $a>0$ จากโจทย์ได้
$n^2+1>(t_1+t_2+...+t_n)(\frac{1}{t_1}+\frac{1}{t_2}+...+\frac{1}{t_n})\geq n+\frac{2t_1}{\sqrt{t_{2}t_{3}}}+\frac{2\sqrt{t_{2}t_{3}}}{t_1}+2[\binom{n}{2}-2]=2a+\frac{2}{a}+n^2-4$
$\therefore 2a+\frac{2}{a}-5< 0$ นั่นคือ $2a^2-5a+2<0$ จะได้ $\frac{1}{2}<a<2$
นั่นคือ $\frac{t_1}{\sqrt{t_{2}t_{3}}}=a<2$
$\therefore t_1<2\sqrt{t_{2}t_{3}}$ แต่จาก $2\sqrt{t_{2}t_{3}}\leq t_2+t_3$
ฉะนั้น $t_1<t_2+t_3$

tatari/nightmare 29 สิงหาคม 2007 20:57

39.Let x,y,zbe positive real numbers such that $xyz\geq 1$,Prove that $$\frac{x^5-x^2}{x^5+y^2+z^2}+\frac{y^5-y^2}{y^5+z^2+x^2}+\frac{z^5-z^2}{z^5+x^2+y^2}\geq 0$$:blood:


เวลาที่แสดงทั้งหมด เป็นเวลาที่ประเทศไทย (GMT +7) ขณะนี้เป็นเวลา 07:37

Powered by vBulletin® Copyright ©2000 - 2024, Jelsoft Enterprises Ltd.
Modified by Jetsada Karnpracha